Những câu hỏi liên quan
hanvu
Xem chi tiết
Trần Phúc Khang
13 tháng 7 2019 lúc 15:35

A

Áp dụng BĐT cosi ta có 

\(\sqrt{\left(2x-1\right).1}\le\frac{2x-1+1}{2}=x\)

\(x\sqrt{5-4x^2}\le\frac{x^2+5-4x^2}{2}=\frac{-3x^2+5}{2}\)

Khi đó 

\(A\le3x+\frac{-3x^2+5}{2}=\frac{-3x^2+6x+5}{2}=\frac{-3\left(x-1\right)^2}{2}+4\le4\)

MaxA=4 khi \(\hept{\begin{cases}2x-1=1\\x^2=5-4x^2\\x=1\end{cases}\Rightarrow}x=1\)

Bình luận (0)
Trần Phúc Khang
13 tháng 7 2019 lúc 15:42

B

Áp dụng BĐT cosi ta có :

\(x^2+y^2+z^2\ge\frac{1}{3}\left(x+y+z\right)^2\)

=> \(x+y+z\le\sqrt{3\left(x^2+y^2+z^2\right)}\)

=> \(B\le\frac{xyz.\left(\sqrt{3\left(x^2+y^2+z^2\right)}+\sqrt{x^2+y^2+z^2}\right)}{\left(x^2+y^2+z^2\right)\left(xy+yz+xz\right)}=\frac{xyz.\left(\sqrt{3}+1\right)}{\left(xy+yz+xz\right)\sqrt{x^2+y^2+z^2}}\)

Lại có \(x^2+y^2+z^2\ge3\sqrt[3]{x^2y^2z^2}\)\(xy+yz+xz\ge3\sqrt[3]{x^2y^2z^2}\)

=> \(\sqrt{x^2+y^2+z^2}\left(xy+yz+xz\right)\ge3\sqrt[3]{x^2y^2z^2}.\sqrt{3\sqrt[3]{x^2y^2z^2}}=3\sqrt{3}.xyz\)

=> \(B\le\frac{\sqrt{3}+1}{3\sqrt{3}}=\frac{3+\sqrt{3}}{9}\)

\(MaxB=\frac{3+\sqrt{3}}{9}\)khi x=y=z

Bình luận (0)
Huy Anh Lê
Xem chi tiết
zZz Nguyễn Quang Duy oOo
28 tháng 4 2016 lúc 17:11

ko làm đâu

Bình luận (0)
Thân Gia Bảo
28 tháng 4 2016 lúc 18:05

Huhu

tui

moi

hoc

lop

5

chua

bit

lam

lop

9

kho

qua

hihi

Bình luận (0)
SKT_ Lạnh _ Lùng
28 tháng 4 2016 lúc 18:12



 

 
 HONG BIET LAM 
?
?
?
?
?
?
?
?
?
?
??

??

??
?
?
?
?
?
?
?
?
?
?
?
?
?
?
?
?
?
??
?
?
?
?
?
?
?
?
?
?
?
?
?
?
?
?

Bình luận (0)
Đen đủi mất cái nik
Xem chi tiết
Nguyễn Tiến Đức
10 tháng 9 2018 lúc 19:41

tự ra câu hởi tự trả lời à bạn

Bình luận (0)
Đen đủi mất cái nik
10 tháng 9 2018 lúc 19:44

tại tui trả lời bài này cho 1 bạn ở trên facebook nên phải chụp màn hình lại nên làm v á

Bình luận (0)
Phương Linh
Xem chi tiết
Mr Lazy
7 tháng 11 2015 lúc 23:15

Côsi: \(\sqrt{x\left(y+z\right)}=\frac{1}{2\sqrt{2}}.2.\sqrt{2x}.\sqrt{y+z}\le\frac{1}{2\sqrt{2}}\left(2x+y+z\right)\)

\(\Rightarrow\frac{1}{\sqrt{x\left(y+z\right)}}\ge\frac{2\sqrt{2}}{2x+y+z}\)

Tương tự các cái kia.

\(\Rightarrow VT\ge2\sqrt{2}\left(\frac{1}{2x+y+z}+\frac{1}{2y+z+x}+\frac{1}{2z+x+y}\right)\)

\(\ge2\sqrt{2}.\frac{9}{2x+y+z+2y+z+x+2z+x+y}=\frac{18\sqrt{2}}{4\left(x+y+z\right)}=\frac{1}{4}\)

Bình luận (0)
Mr Lazy
7 tháng 11 2015 lúc 22:56

\(\sum\frac{1}{\sqrt{x\left(y+z\right)}}=\sum\frac{\sqrt{2}}{\sqrt{2x}.\sqrt{y+z}}\ge\sum\frac{2\sqrt{2}}{2x+y+z}\ge2\sqrt{2}.\frac{9}{\sum\left(2x+y+z\right)}=\frac{18\sqrt{2}}{4\left(x+y+z\right)}=\frac{1}{4}\)

Bình luận (0)
Thanh Tâm
Xem chi tiết
hoàng minh chính
Xem chi tiết
Nguyễn Việt Lâm
9 tháng 4 2022 lúc 17:56

Ta có:

\(1.\sqrt{1+x^2}+1.\sqrt{2x}\le\sqrt{\left(1+1\right)\left(1+x^2+2x\right)}=\sqrt{2}\left(x+1\right)\)

Tương tự:

\(\sqrt{1+y^2}+\sqrt{2y}\le\sqrt{2}\left(y+1\right)\) ; \(\sqrt{1+z^2}+\sqrt{2z}\le\sqrt{2}\left(z+1\right)\)

Cộng vế:

\(P\le\sqrt{2}\left(x+y+z+3\right)+\left(2-\sqrt{2}\right)\left(x+y+z\right)\le\sqrt{2}\left(3+3\right)+\left(2-\sqrt{2}\right).3=6+3\sqrt{2}\)

\(P_{max}=6+3\sqrt{2}\) khi \(x=y=z=1\)

Bình luận (0)
Easylove
Xem chi tiết
Dương
Xem chi tiết
Kiệt Nguyễn
13 tháng 3 2021 lúc 7:36

Theo giả thiết xy + yz + zx = 1 nên ta có: \(VT=\frac{1}{1+x^2}+\frac{1}{1+y^2}+\frac{1}{1+z^2}=\frac{1}{xy+yz+zx+x^2}+\frac{1}{xy+yz+zx+y^2}+\frac{1}{xy+yz+zx+z^2}=\frac{1}{\left(x+y\right)\left(x+z\right)}+\frac{1}{\left(y+x\right)\left(y+z\right)}+\frac{1}{\left(z+x\right)\left(z+y\right)}=\frac{2\left(x+y+z\right)}{\left(x+y\right)\left(y+z\right)\left(z+x\right)}\)Theo bất đẳng thức Cauchy-Schwarz: \(\left(\frac{x}{\sqrt{1+x^2}}+\frac{y}{\sqrt{1+y^2}}+\frac{z}{\sqrt{1+z^2}}\right)^2\le\left(x+y+z\right)\left(\frac{x}{1+x^2}+\frac{y}{1+y^2}+\frac{z}{1+z^2}\right)=\left(x+y+z\right)\left(\frac{x}{\left(x+y\right)\left(x+z\right)}+\frac{y}{\left(y+z\right)\left(y+x\right)}+\frac{z}{\left(z+x\right)\left(z+y\right)}\right)=\frac{2\left(x+y+z\right)\left(xy+yz+zx\right)}{\left(x+y\right)\left(y+z\right)\left(z+x\right)}=\frac{2\left(x+y+z\right)}{\left(x+y\right)\left(y+z\right)\left(z+x\right)}\)\(\Rightarrow\frac{2}{3}\left(\frac{x}{\sqrt{1+x^2}}+\frac{y}{\sqrt{1+y^2}}+\frac{z}{\sqrt{1+z^2}}\right)^3\le\frac{4\left(x+y+z\right)}{3\left(x+y\right)\left(y+z\right)\left(z+x\right)}\left(\frac{x}{\sqrt{1+x^2}}+\frac{y}{\sqrt{1+y^2}}+\frac{z}{\sqrt{1+z^2}}\right)\)Ta cần chứng minh: \(\frac{2\left(x+y+z\right)}{\left(x+y\right)\left(y+z\right)\left(z+x\right)}\ge\frac{4\left(x+y+z\right)}{3\left(x+y\right)\left(y+z\right)\left(z+x\right)}\left(\frac{x}{\sqrt{1+x^2}}+\frac{y}{\sqrt{1+y^2}}+\frac{z}{\sqrt{1+z^2}}\right)\)

hay \(\frac{x}{\sqrt{1+x^2}}+\frac{y}{\sqrt{1+y^2}}+\frac{z}{\sqrt{1+z^2}}\le\frac{3}{2}\)

Bất đẳng thức cuối đúng theo AM - GM do: \(\frac{x}{\sqrt{1+x^2}}+\frac{y}{\sqrt{1+y^2}}+\frac{z}{\sqrt{1+z^2}}=\sqrt{\frac{x}{x+y}.\frac{x}{x+z}}+\sqrt{\frac{y}{y+z}.\frac{y}{x+y}}+\sqrt{\frac{z}{z+x}.\frac{z}{z+y}}\le\frac{\left(\frac{x}{x+y}+\frac{x}{x+z}\right)+\left(\frac{y}{y+z}+\frac{y}{x+y}\right)+\left(\frac{z}{z+x}+\frac{z}{z+y}\right)}{2}=\frac{3}{2}\)Đẳng thức xảy ra khi \(x=y=z=\frac{1}{\sqrt{3}}\)

Bình luận (0)
 Khách vãng lai đã xóa
Vũ Thu Mai
Xem chi tiết